آيا شما مايل به راه افتادن تاپيك ماراتن نامساوي مي باشي


  • مجموع رای دهندگان
    339

Dadgarnia

New Member
ارسال ها
1,350
لایک ها
1,127
امتیاز
0
پاسخ : ماراتن نامساوي

من فقط اثبات اينكه k كوچكتر يا مساوي ٣ هست رو اينجا مي نويسم تا حداقل مشخص بشه چي رو مي خوايم اثبات كنيم:
اگه دو تا يك بديم بدست مياد:
حالا اگه طرفين وسطين كنيم و يه كم ساده كنيم بدست مياد:


اگه a يك نباشه بدست مياد
كه حداقل اين عبارت هم برابر با ٣ هست و اگه a يك باشه حالت تساوي رخ ميده.
 

sepidfekr

New Member
ارسال ها
711
لایک ها
637
امتیاز
0
پاسخ : ماراتن نامساوي

دوستان بهتر نیست سوال بعد رو بزاریم؟؟؟چون واقعا ماراتن خوابیده:4::4::4:

---- دو نوشته به هم متصل شده است ----

سوال بعد:
 

m-saghaei

New Member
ارسال ها
338
لایک ها
258
امتیاز
0
پاسخ : ماراتن نامساوي

دوستان بهتر نیست سوال بعد رو بزاریم؟؟؟چون واقعا ماراتن خوابیده:4::4::4:

---- دو نوشته به هم متصل شده است ----

سوال بعد:
بهتر بود میزاشتید سوال قبل حل شه بعد سوال بعد رو بزارید.
 

sepidfekr

New Member
ارسال ها
711
لایک ها
637
امتیاز
0
پاسخ : ماراتن نامساوي

بهتر بود میزاشتید سوال قبل حل شه بعد سوال بعد رو بزارید.
من از چند تا از دوستان پرسیدم گفتن بزارم انشاالله سوال قبلی رو آقای شریفی میان و حل میکنن;)
 

MGH000

New Member
ارسال ها
209
لایک ها
219
امتیاز
0
پاسخ : ماراتن نامساوي

گه اشتباه نکرده باشم اینه جواب سوال:


یک حسابی هندسی در مخرج طرف کوچکتر میزنیم


اگر k یک نبود:


آیا

آیا
که به وضوح درست است حال:


حال میدانیم عبارت های ترکیبی کوچکترند.ب عنوان مثال اگر a,b,c را 1و2و3 قرار دهیم واضحا طرفی که باید کوچکتر باشد بزرگتر میشود درنتیجه k=1 است.
حال حالت k=1 نیز به راحتی اثبات میشود.
 
آخرین ویرایش توسط مدیر

Dadgarnia

New Member
ارسال ها
1,350
لایک ها
1,127
امتیاز
0
پاسخ : ماراتن نامساوي

دوستان بهتر نیست سوال بعد رو بزاریم؟؟؟چون واقعا ماراتن خوابیده:4::4::4:

---- دو نوشته به هم متصل شده است ----

سوال بعد:
مي تونيم فرض كنيم
حالا اگه اين رابطه رو توي صورت سوال قرار بديم و يكم ساده كنيم بدست مياد:

 

aras2213

New Member
ارسال ها
216
لایک ها
228
امتیاز
0
پاسخ : ماراتن نامساوي

الآن درست شد دیگه؟

---- دو نوشته به هم متصل شده است ----

اینم سوال بعد:

ثابت کنید:
که با استفاده از نامساوی کوشی کافیه ثابت کنیم:




که با توجه به فرض مساله و نامساوی شور درست است.
 

AHZolfaghari

Well-Known Member
ارسال ها
935
لایک ها
1,654
امتیاز
93
پاسخ : ماراتن نامساوي

البته پاسخ mgh000 برای اون سوال نادرست است . به فکر کردن روی اون سوال ادامه بدید !
 

Dadgarnia

New Member
ارسال ها
1,350
لایک ها
1,127
امتیاز
0
پاسخ : ماراتن نامساوي

البته پاسخ mgh000 برای اون سوال نادرست است . به فکر کردن روی اون سوال ادامه بدید !
ميشه بگيد كجاش اشكال داره؟ مي دونم كه آقاي شريفي گفتن ولي شايد منظور ايشون يه چيز ديگه بوده. روند صحيح اثبات آقاي MGH000 اينجوريه كه اول براي k=1 اثبات كنيم و بعدش مي تونيم اون نامساوي ها رو با فرض k>1 بنويسيم.
 

AHZolfaghari

Well-Known Member
ارسال ها
935
لایک ها
1,654
امتیاز
93
پاسخ : ماراتن نامساوي

ميشه بگيد كجاش اشكال داره؟ مي دونم كه آقاي شريفي گفتن ولي شايد منظور ايشون يه چيز ديگه بوده. روند صحيح اثبات آقاي MGH000 اينجوريه كه اول براي k=1 اثبات كنيم و بعدش مي تونيم اون نامساوي ها رو با فرض k>1 بنويسيم.
همون تیکش که تو مخرج حسابی هندسی زده باعث وارد شدن اشکال به راه حله .
 

Dadgarnia

New Member
ارسال ها
1,350
لایک ها
1,127
امتیاز
0
پاسخ : ماراتن نامساوي

همون تیکش که تو مخرج حسابی هندسی زده باعث وارد شدن اشکال به راه حله .
شما منظورتون بعد از اين قسمته ديگه:
ما اگه بخوايم روي مخرج سمت چپ حسابي هندسي بزنيم بايد صورت كسرمون مثبت باشه و مخرج كسرمون هم همين طور پس با فرض
كسرمون شرايط مورد نظرو داره و مي تونيم حسابي هندسي بزنيم. در صورتي كه اگه ما فرض
رو نداشته باشيم نمي تونيم اين كارو انجام بديم و احتمالا منظور آقاي شريفي هم همين بوده.
 

m-saghaei

New Member
ارسال ها
338
لایک ها
258
امتیاز
0
پاسخ : ماراتن نامساوي

تا دو روز دیگه رو اون سوال فکر کنید اگه حل نشد سوال بعد رو بزارین لطفا.(مجبوریم به آقا شریفی بگیم بیان حل کنن!)
 

m-saghaei

New Member
ارسال ها
338
لایک ها
258
امتیاز
0
پاسخ : ماراتن نامساوي

سوال بعد(آسون):
اگر
و
سه عدد حقیقی مثبت باشند ثابت کنید:


(هر کی حل کرد سوال بعد رو خودش بزاره!)
 

AHZolfaghari

Well-Known Member
ارسال ها
935
لایک ها
1,654
امتیاز
93
پاسخ : ماراتن نامساوي

سوال بعد(آسون):
اگر
و
سه عدد حقیقی مثبت باشند ثابت کنید:


(هر کی حل کرد سوال بعد رو خودش بزاره!)

پس

با توجه به کوشی

اما با حسابی هندسی روی ab+bc+ca می فهمیم که


پس

که مقصود سوال است
 

m-saghaei

New Member
ارسال ها
338
لایک ها
258
امتیاز
0
پاسخ : ماراتن نامساوي

سوال بعد:
a,b,c سه تا عدد بزرگتر از صفر هستن و داریم
ثابت کنید:

 
بالا